What are the eigenvalues of the given matrix $M$












2












$begingroup$


I am learning linear algebra and now I'm in eigenvalues and eigenvectors part of it. there is a question that I can't solve it or any idea that I have is hard and nasty. I think this question must have a trick that I am not familiar with it because I'm new to eigenvalues and eigenvectors.



the question is this:




Let $M in Bbb R^{ntimes n}$ and real numbers $a_1$ to $a_n$
and every $m_{ij} = frac{a_i}{a_j}$, so:
$$ M = begin{pmatrix}1&cdots&frac{a_1}{a_n}\vdots&ddots&vdots\frac{a_n}{a_1}&cdots&1end{pmatrix} $$
find all eigenvalues.




any help would be appreciated.










share|cite|improve this question











$endgroup$












  • $begingroup$
    But you are saying from the start that the eigenvalues are the $lambda_i$'s. Are you sure about that?
    $endgroup$
    – José Carlos Santos
    Dec 20 '18 at 19:51










  • $begingroup$
    A matrix of size $ntimes n$ can't have more than $n$ distinct eigenvalues. So your question makes no sense as it is written right now.
    $endgroup$
    – Mark
    Dec 20 '18 at 19:52












  • $begingroup$
    sorryyyy! I'm going to edit it
    $endgroup$
    – Peyman mohseni kiasari
    Dec 20 '18 at 19:53






  • 9




    $begingroup$
    Note that all columns are scalar multiples of the first column. Thus this matrix has rank $1$, and there is only one nonzero eigenvalue.
    $endgroup$
    – Robert Israel
    Dec 20 '18 at 19:57






  • 1




    $begingroup$
    @Damien Seems more fruitful to look at the vector $(a_1, a_2, ldots, a_n)$.
    $endgroup$
    – Bungo
    Dec 20 '18 at 20:18
















2












$begingroup$


I am learning linear algebra and now I'm in eigenvalues and eigenvectors part of it. there is a question that I can't solve it or any idea that I have is hard and nasty. I think this question must have a trick that I am not familiar with it because I'm new to eigenvalues and eigenvectors.



the question is this:




Let $M in Bbb R^{ntimes n}$ and real numbers $a_1$ to $a_n$
and every $m_{ij} = frac{a_i}{a_j}$, so:
$$ M = begin{pmatrix}1&cdots&frac{a_1}{a_n}\vdots&ddots&vdots\frac{a_n}{a_1}&cdots&1end{pmatrix} $$
find all eigenvalues.




any help would be appreciated.










share|cite|improve this question











$endgroup$












  • $begingroup$
    But you are saying from the start that the eigenvalues are the $lambda_i$'s. Are you sure about that?
    $endgroup$
    – José Carlos Santos
    Dec 20 '18 at 19:51










  • $begingroup$
    A matrix of size $ntimes n$ can't have more than $n$ distinct eigenvalues. So your question makes no sense as it is written right now.
    $endgroup$
    – Mark
    Dec 20 '18 at 19:52












  • $begingroup$
    sorryyyy! I'm going to edit it
    $endgroup$
    – Peyman mohseni kiasari
    Dec 20 '18 at 19:53






  • 9




    $begingroup$
    Note that all columns are scalar multiples of the first column. Thus this matrix has rank $1$, and there is only one nonzero eigenvalue.
    $endgroup$
    – Robert Israel
    Dec 20 '18 at 19:57






  • 1




    $begingroup$
    @Damien Seems more fruitful to look at the vector $(a_1, a_2, ldots, a_n)$.
    $endgroup$
    – Bungo
    Dec 20 '18 at 20:18














2












2








2


0



$begingroup$


I am learning linear algebra and now I'm in eigenvalues and eigenvectors part of it. there is a question that I can't solve it or any idea that I have is hard and nasty. I think this question must have a trick that I am not familiar with it because I'm new to eigenvalues and eigenvectors.



the question is this:




Let $M in Bbb R^{ntimes n}$ and real numbers $a_1$ to $a_n$
and every $m_{ij} = frac{a_i}{a_j}$, so:
$$ M = begin{pmatrix}1&cdots&frac{a_1}{a_n}\vdots&ddots&vdots\frac{a_n}{a_1}&cdots&1end{pmatrix} $$
find all eigenvalues.




any help would be appreciated.










share|cite|improve this question











$endgroup$




I am learning linear algebra and now I'm in eigenvalues and eigenvectors part of it. there is a question that I can't solve it or any idea that I have is hard and nasty. I think this question must have a trick that I am not familiar with it because I'm new to eigenvalues and eigenvectors.



the question is this:




Let $M in Bbb R^{ntimes n}$ and real numbers $a_1$ to $a_n$
and every $m_{ij} = frac{a_i}{a_j}$, so:
$$ M = begin{pmatrix}1&cdots&frac{a_1}{a_n}\vdots&ddots&vdots\frac{a_n}{a_1}&cdots&1end{pmatrix} $$
find all eigenvalues.




any help would be appreciated.







linear-algebra matrices eigenvalues-eigenvectors






share|cite|improve this question















share|cite|improve this question













share|cite|improve this question




share|cite|improve this question








edited Dec 20 '18 at 20:17









mechanodroid

28.9k62648




28.9k62648










asked Dec 20 '18 at 19:49









Peyman mohseni kiasariPeyman mohseni kiasari

13711




13711












  • $begingroup$
    But you are saying from the start that the eigenvalues are the $lambda_i$'s. Are you sure about that?
    $endgroup$
    – José Carlos Santos
    Dec 20 '18 at 19:51










  • $begingroup$
    A matrix of size $ntimes n$ can't have more than $n$ distinct eigenvalues. So your question makes no sense as it is written right now.
    $endgroup$
    – Mark
    Dec 20 '18 at 19:52












  • $begingroup$
    sorryyyy! I'm going to edit it
    $endgroup$
    – Peyman mohseni kiasari
    Dec 20 '18 at 19:53






  • 9




    $begingroup$
    Note that all columns are scalar multiples of the first column. Thus this matrix has rank $1$, and there is only one nonzero eigenvalue.
    $endgroup$
    – Robert Israel
    Dec 20 '18 at 19:57






  • 1




    $begingroup$
    @Damien Seems more fruitful to look at the vector $(a_1, a_2, ldots, a_n)$.
    $endgroup$
    – Bungo
    Dec 20 '18 at 20:18


















  • $begingroup$
    But you are saying from the start that the eigenvalues are the $lambda_i$'s. Are you sure about that?
    $endgroup$
    – José Carlos Santos
    Dec 20 '18 at 19:51










  • $begingroup$
    A matrix of size $ntimes n$ can't have more than $n$ distinct eigenvalues. So your question makes no sense as it is written right now.
    $endgroup$
    – Mark
    Dec 20 '18 at 19:52












  • $begingroup$
    sorryyyy! I'm going to edit it
    $endgroup$
    – Peyman mohseni kiasari
    Dec 20 '18 at 19:53






  • 9




    $begingroup$
    Note that all columns are scalar multiples of the first column. Thus this matrix has rank $1$, and there is only one nonzero eigenvalue.
    $endgroup$
    – Robert Israel
    Dec 20 '18 at 19:57






  • 1




    $begingroup$
    @Damien Seems more fruitful to look at the vector $(a_1, a_2, ldots, a_n)$.
    $endgroup$
    – Bungo
    Dec 20 '18 at 20:18
















$begingroup$
But you are saying from the start that the eigenvalues are the $lambda_i$'s. Are you sure about that?
$endgroup$
– José Carlos Santos
Dec 20 '18 at 19:51




$begingroup$
But you are saying from the start that the eigenvalues are the $lambda_i$'s. Are you sure about that?
$endgroup$
– José Carlos Santos
Dec 20 '18 at 19:51












$begingroup$
A matrix of size $ntimes n$ can't have more than $n$ distinct eigenvalues. So your question makes no sense as it is written right now.
$endgroup$
– Mark
Dec 20 '18 at 19:52






$begingroup$
A matrix of size $ntimes n$ can't have more than $n$ distinct eigenvalues. So your question makes no sense as it is written right now.
$endgroup$
– Mark
Dec 20 '18 at 19:52














$begingroup$
sorryyyy! I'm going to edit it
$endgroup$
– Peyman mohseni kiasari
Dec 20 '18 at 19:53




$begingroup$
sorryyyy! I'm going to edit it
$endgroup$
– Peyman mohseni kiasari
Dec 20 '18 at 19:53




9




9




$begingroup$
Note that all columns are scalar multiples of the first column. Thus this matrix has rank $1$, and there is only one nonzero eigenvalue.
$endgroup$
– Robert Israel
Dec 20 '18 at 19:57




$begingroup$
Note that all columns are scalar multiples of the first column. Thus this matrix has rank $1$, and there is only one nonzero eigenvalue.
$endgroup$
– Robert Israel
Dec 20 '18 at 19:57




1




1




$begingroup$
@Damien Seems more fruitful to look at the vector $(a_1, a_2, ldots, a_n)$.
$endgroup$
– Bungo
Dec 20 '18 at 20:18




$begingroup$
@Damien Seems more fruitful to look at the vector $(a_1, a_2, ldots, a_n)$.
$endgroup$
– Bungo
Dec 20 '18 at 20:18










3 Answers
3






active

oldest

votes


















3












$begingroup$

Another way to do this is by noting that if $mathbf{a} = (a_1, dots, a_n)^top$ and $mathbf{b} = (1/a_1, dots, 1/a_n)^top$, then
$$
M = mathbf{a} mathbf{b}^top,
$$

where $mathbf{b}^top$ denotes the transpose of $mathbf{b}$. The rank of $M$ is therefore 1 (can you see why?), meaning that only one eigenvalue is non-zero. This eigenvalue is found by considering
$$
M mathbf{a} = (mathbf{a} mathbf{b}^top) mathbf{a} = mathbf{a} (mathbf{b}^top mathbf{a}) = n mathbf{a},
$$

i.e. the final eigenvalue is $n$.






share|cite|improve this answer











$endgroup$













  • $begingroup$
    short and nice. sorry if my question is silly but how do you mult two vectors in this way? I just know that mult of two vectors is a number. this way of mult is not in the book yet.
    $endgroup$
    – Peyman mohseni kiasari
    Dec 20 '18 at 20:28










  • $begingroup$
    @Peyman, you may interpret a vector as an $ntimes 1$ matrix.
    $endgroup$
    – Decaf-Math
    Dec 20 '18 at 20:30












  • $begingroup$
    You multiply them by normal matrix multiplication. Note that the number of columns of $mathbf{a}$ (i.e. one) is equal to the number of rows of $mathbf{b}^top$ (also one).
    $endgroup$
    – ekkilop
    Dec 20 '18 at 20:32





















2












$begingroup$

To find the eigenvalues, we are calculating the zeroes of the characteristic polynomial of $M$.



$$0= det(M - lambda I) = begin{vmatrix} 1-lambda &frac{a_1}{a_2} & cdots & frac{a_1}{a_n} \
frac{a_2}{a_1} & 1-lambda & cdots & frac{a_2}{a_n} \
vdots & vdots & ddots & vdots \
frac{a_n}{a_1} & frac{a_n}{a_2} & cdots & 1-lambda
end{vmatrix}$$



Since $a_1, ldots, a_n ne 0$, we can multiply $j$-th column by $a_j$ for $j = 1, ldots, n$ to obtain:



$$0 = begin{vmatrix} a_1(1-lambda) & a_1 & cdots & a_1 \
a_2 & a_2(1-lambda) & cdots & a_2 \
vdots & vdots & ddots & vdots \
a_n & a_n & cdots & a_n(1-lambda)
end{vmatrix}$$



Now divide $i$-th row by $a_i$ for $i =1, ldots, n$ to obtain



begin{align}
0 &= begin{vmatrix} 1-lambda & 1 & cdots & 1 \
1 & 1-lambda & cdots & 1 \
vdots & vdots & ddots & vdots \
1 & 1 & cdots & 1-lambda
end{vmatrix} \
&= begin{vmatrix} 1-lambda & 1 & cdots & 1 \
lambda & -lambda & cdots & 0 \
vdots & vdots & ddots & vdots \
lambda & 0 & cdots & -lambda
end{vmatrix} \
&= begin{vmatrix} n-lambda & 1 & cdots & 1 \
0 & -lambda & cdots & 0 \
vdots & vdots & ddots & vdots \
0& 0 & cdots & -lambda
end{vmatrix} \
&= (n-lambda)(-lambda)^{n-1}
end{align}



so the eigenvalues are $0$ and $n$.






share|cite|improve this answer











$endgroup$









  • 1




    $begingroup$
    I think it is wrong. let a1=1 and a2 =1 then M is [[1,1],[1,1]]. but the eigenvalues of M are 0 and 2
    $endgroup$
    – Peyman mohseni kiasari
    Dec 20 '18 at 20:10










  • $begingroup$
    @Peyman Whoops, you are right. The eigenvalues are $0$ and $n$.
    $endgroup$
    – mechanodroid
    Dec 20 '18 at 20:15



















1












$begingroup$

We can write your matrix as $M = DJD^{-1}$, where
$$
D = pmatrix{a_1\ & ddots \ && a_n}, quad
J = pmatrix{1 & cdots & 1\ vdots & ddots & vdots \ 1 & cdots & 1}
$$

So, $M$ is similar to $J$. $J$ is a rank $1$ symmetric matrix, so it's only non-zero eigenvalue will be $operatorname{tr}(J) = n$.



We could also recognize that $M$ has rank $1$ as Robert did in his comment.






share|cite|improve this answer









$endgroup$














    Your Answer





    StackExchange.ifUsing("editor", function () {
    return StackExchange.using("mathjaxEditing", function () {
    StackExchange.MarkdownEditor.creationCallbacks.add(function (editor, postfix) {
    StackExchange.mathjaxEditing.prepareWmdForMathJax(editor, postfix, [["$", "$"], ["\\(","\\)"]]);
    });
    });
    }, "mathjax-editing");

    StackExchange.ready(function() {
    var channelOptions = {
    tags: "".split(" "),
    id: "69"
    };
    initTagRenderer("".split(" "), "".split(" "), channelOptions);

    StackExchange.using("externalEditor", function() {
    // Have to fire editor after snippets, if snippets enabled
    if (StackExchange.settings.snippets.snippetsEnabled) {
    StackExchange.using("snippets", function() {
    createEditor();
    });
    }
    else {
    createEditor();
    }
    });

    function createEditor() {
    StackExchange.prepareEditor({
    heartbeatType: 'answer',
    autoActivateHeartbeat: false,
    convertImagesToLinks: true,
    noModals: true,
    showLowRepImageUploadWarning: true,
    reputationToPostImages: 10,
    bindNavPrevention: true,
    postfix: "",
    imageUploader: {
    brandingHtml: "Powered by u003ca class="icon-imgur-white" href="https://imgur.com/"u003eu003c/au003e",
    contentPolicyHtml: "User contributions licensed under u003ca href="https://creativecommons.org/licenses/by-sa/3.0/"u003ecc by-sa 3.0 with attribution requiredu003c/au003e u003ca href="https://stackoverflow.com/legal/content-policy"u003e(content policy)u003c/au003e",
    allowUrls: true
    },
    noCode: true, onDemand: true,
    discardSelector: ".discard-answer"
    ,immediatelyShowMarkdownHelp:true
    });


    }
    });














    draft saved

    draft discarded


















    StackExchange.ready(
    function () {
    StackExchange.openid.initPostLogin('.new-post-login', 'https%3a%2f%2fmath.stackexchange.com%2fquestions%2f3047908%2fwhat-are-the-eigenvalues-of-the-given-matrix-m%23new-answer', 'question_page');
    }
    );

    Post as a guest















    Required, but never shown

























    3 Answers
    3






    active

    oldest

    votes








    3 Answers
    3






    active

    oldest

    votes









    active

    oldest

    votes






    active

    oldest

    votes









    3












    $begingroup$

    Another way to do this is by noting that if $mathbf{a} = (a_1, dots, a_n)^top$ and $mathbf{b} = (1/a_1, dots, 1/a_n)^top$, then
    $$
    M = mathbf{a} mathbf{b}^top,
    $$

    where $mathbf{b}^top$ denotes the transpose of $mathbf{b}$. The rank of $M$ is therefore 1 (can you see why?), meaning that only one eigenvalue is non-zero. This eigenvalue is found by considering
    $$
    M mathbf{a} = (mathbf{a} mathbf{b}^top) mathbf{a} = mathbf{a} (mathbf{b}^top mathbf{a}) = n mathbf{a},
    $$

    i.e. the final eigenvalue is $n$.






    share|cite|improve this answer











    $endgroup$













    • $begingroup$
      short and nice. sorry if my question is silly but how do you mult two vectors in this way? I just know that mult of two vectors is a number. this way of mult is not in the book yet.
      $endgroup$
      – Peyman mohseni kiasari
      Dec 20 '18 at 20:28










    • $begingroup$
      @Peyman, you may interpret a vector as an $ntimes 1$ matrix.
      $endgroup$
      – Decaf-Math
      Dec 20 '18 at 20:30












    • $begingroup$
      You multiply them by normal matrix multiplication. Note that the number of columns of $mathbf{a}$ (i.e. one) is equal to the number of rows of $mathbf{b}^top$ (also one).
      $endgroup$
      – ekkilop
      Dec 20 '18 at 20:32


















    3












    $begingroup$

    Another way to do this is by noting that if $mathbf{a} = (a_1, dots, a_n)^top$ and $mathbf{b} = (1/a_1, dots, 1/a_n)^top$, then
    $$
    M = mathbf{a} mathbf{b}^top,
    $$

    where $mathbf{b}^top$ denotes the transpose of $mathbf{b}$. The rank of $M$ is therefore 1 (can you see why?), meaning that only one eigenvalue is non-zero. This eigenvalue is found by considering
    $$
    M mathbf{a} = (mathbf{a} mathbf{b}^top) mathbf{a} = mathbf{a} (mathbf{b}^top mathbf{a}) = n mathbf{a},
    $$

    i.e. the final eigenvalue is $n$.






    share|cite|improve this answer











    $endgroup$













    • $begingroup$
      short and nice. sorry if my question is silly but how do you mult two vectors in this way? I just know that mult of two vectors is a number. this way of mult is not in the book yet.
      $endgroup$
      – Peyman mohseni kiasari
      Dec 20 '18 at 20:28










    • $begingroup$
      @Peyman, you may interpret a vector as an $ntimes 1$ matrix.
      $endgroup$
      – Decaf-Math
      Dec 20 '18 at 20:30












    • $begingroup$
      You multiply them by normal matrix multiplication. Note that the number of columns of $mathbf{a}$ (i.e. one) is equal to the number of rows of $mathbf{b}^top$ (also one).
      $endgroup$
      – ekkilop
      Dec 20 '18 at 20:32
















    3












    3








    3





    $begingroup$

    Another way to do this is by noting that if $mathbf{a} = (a_1, dots, a_n)^top$ and $mathbf{b} = (1/a_1, dots, 1/a_n)^top$, then
    $$
    M = mathbf{a} mathbf{b}^top,
    $$

    where $mathbf{b}^top$ denotes the transpose of $mathbf{b}$. The rank of $M$ is therefore 1 (can you see why?), meaning that only one eigenvalue is non-zero. This eigenvalue is found by considering
    $$
    M mathbf{a} = (mathbf{a} mathbf{b}^top) mathbf{a} = mathbf{a} (mathbf{b}^top mathbf{a}) = n mathbf{a},
    $$

    i.e. the final eigenvalue is $n$.






    share|cite|improve this answer











    $endgroup$



    Another way to do this is by noting that if $mathbf{a} = (a_1, dots, a_n)^top$ and $mathbf{b} = (1/a_1, dots, 1/a_n)^top$, then
    $$
    M = mathbf{a} mathbf{b}^top,
    $$

    where $mathbf{b}^top$ denotes the transpose of $mathbf{b}$. The rank of $M$ is therefore 1 (can you see why?), meaning that only one eigenvalue is non-zero. This eigenvalue is found by considering
    $$
    M mathbf{a} = (mathbf{a} mathbf{b}^top) mathbf{a} = mathbf{a} (mathbf{b}^top mathbf{a}) = n mathbf{a},
    $$

    i.e. the final eigenvalue is $n$.







    share|cite|improve this answer














    share|cite|improve this answer



    share|cite|improve this answer








    edited Dec 20 '18 at 20:35

























    answered Dec 20 '18 at 20:23









    ekkilopekkilop

    1,736519




    1,736519












    • $begingroup$
      short and nice. sorry if my question is silly but how do you mult two vectors in this way? I just know that mult of two vectors is a number. this way of mult is not in the book yet.
      $endgroup$
      – Peyman mohseni kiasari
      Dec 20 '18 at 20:28










    • $begingroup$
      @Peyman, you may interpret a vector as an $ntimes 1$ matrix.
      $endgroup$
      – Decaf-Math
      Dec 20 '18 at 20:30












    • $begingroup$
      You multiply them by normal matrix multiplication. Note that the number of columns of $mathbf{a}$ (i.e. one) is equal to the number of rows of $mathbf{b}^top$ (also one).
      $endgroup$
      – ekkilop
      Dec 20 '18 at 20:32




















    • $begingroup$
      short and nice. sorry if my question is silly but how do you mult two vectors in this way? I just know that mult of two vectors is a number. this way of mult is not in the book yet.
      $endgroup$
      – Peyman mohseni kiasari
      Dec 20 '18 at 20:28










    • $begingroup$
      @Peyman, you may interpret a vector as an $ntimes 1$ matrix.
      $endgroup$
      – Decaf-Math
      Dec 20 '18 at 20:30












    • $begingroup$
      You multiply them by normal matrix multiplication. Note that the number of columns of $mathbf{a}$ (i.e. one) is equal to the number of rows of $mathbf{b}^top$ (also one).
      $endgroup$
      – ekkilop
      Dec 20 '18 at 20:32


















    $begingroup$
    short and nice. sorry if my question is silly but how do you mult two vectors in this way? I just know that mult of two vectors is a number. this way of mult is not in the book yet.
    $endgroup$
    – Peyman mohseni kiasari
    Dec 20 '18 at 20:28




    $begingroup$
    short and nice. sorry if my question is silly but how do you mult two vectors in this way? I just know that mult of two vectors is a number. this way of mult is not in the book yet.
    $endgroup$
    – Peyman mohseni kiasari
    Dec 20 '18 at 20:28












    $begingroup$
    @Peyman, you may interpret a vector as an $ntimes 1$ matrix.
    $endgroup$
    – Decaf-Math
    Dec 20 '18 at 20:30






    $begingroup$
    @Peyman, you may interpret a vector as an $ntimes 1$ matrix.
    $endgroup$
    – Decaf-Math
    Dec 20 '18 at 20:30














    $begingroup$
    You multiply them by normal matrix multiplication. Note that the number of columns of $mathbf{a}$ (i.e. one) is equal to the number of rows of $mathbf{b}^top$ (also one).
    $endgroup$
    – ekkilop
    Dec 20 '18 at 20:32






    $begingroup$
    You multiply them by normal matrix multiplication. Note that the number of columns of $mathbf{a}$ (i.e. one) is equal to the number of rows of $mathbf{b}^top$ (also one).
    $endgroup$
    – ekkilop
    Dec 20 '18 at 20:32













    2












    $begingroup$

    To find the eigenvalues, we are calculating the zeroes of the characteristic polynomial of $M$.



    $$0= det(M - lambda I) = begin{vmatrix} 1-lambda &frac{a_1}{a_2} & cdots & frac{a_1}{a_n} \
    frac{a_2}{a_1} & 1-lambda & cdots & frac{a_2}{a_n} \
    vdots & vdots & ddots & vdots \
    frac{a_n}{a_1} & frac{a_n}{a_2} & cdots & 1-lambda
    end{vmatrix}$$



    Since $a_1, ldots, a_n ne 0$, we can multiply $j$-th column by $a_j$ for $j = 1, ldots, n$ to obtain:



    $$0 = begin{vmatrix} a_1(1-lambda) & a_1 & cdots & a_1 \
    a_2 & a_2(1-lambda) & cdots & a_2 \
    vdots & vdots & ddots & vdots \
    a_n & a_n & cdots & a_n(1-lambda)
    end{vmatrix}$$



    Now divide $i$-th row by $a_i$ for $i =1, ldots, n$ to obtain



    begin{align}
    0 &= begin{vmatrix} 1-lambda & 1 & cdots & 1 \
    1 & 1-lambda & cdots & 1 \
    vdots & vdots & ddots & vdots \
    1 & 1 & cdots & 1-lambda
    end{vmatrix} \
    &= begin{vmatrix} 1-lambda & 1 & cdots & 1 \
    lambda & -lambda & cdots & 0 \
    vdots & vdots & ddots & vdots \
    lambda & 0 & cdots & -lambda
    end{vmatrix} \
    &= begin{vmatrix} n-lambda & 1 & cdots & 1 \
    0 & -lambda & cdots & 0 \
    vdots & vdots & ddots & vdots \
    0& 0 & cdots & -lambda
    end{vmatrix} \
    &= (n-lambda)(-lambda)^{n-1}
    end{align}



    so the eigenvalues are $0$ and $n$.






    share|cite|improve this answer











    $endgroup$









    • 1




      $begingroup$
      I think it is wrong. let a1=1 and a2 =1 then M is [[1,1],[1,1]]. but the eigenvalues of M are 0 and 2
      $endgroup$
      – Peyman mohseni kiasari
      Dec 20 '18 at 20:10










    • $begingroup$
      @Peyman Whoops, you are right. The eigenvalues are $0$ and $n$.
      $endgroup$
      – mechanodroid
      Dec 20 '18 at 20:15
















    2












    $begingroup$

    To find the eigenvalues, we are calculating the zeroes of the characteristic polynomial of $M$.



    $$0= det(M - lambda I) = begin{vmatrix} 1-lambda &frac{a_1}{a_2} & cdots & frac{a_1}{a_n} \
    frac{a_2}{a_1} & 1-lambda & cdots & frac{a_2}{a_n} \
    vdots & vdots & ddots & vdots \
    frac{a_n}{a_1} & frac{a_n}{a_2} & cdots & 1-lambda
    end{vmatrix}$$



    Since $a_1, ldots, a_n ne 0$, we can multiply $j$-th column by $a_j$ for $j = 1, ldots, n$ to obtain:



    $$0 = begin{vmatrix} a_1(1-lambda) & a_1 & cdots & a_1 \
    a_2 & a_2(1-lambda) & cdots & a_2 \
    vdots & vdots & ddots & vdots \
    a_n & a_n & cdots & a_n(1-lambda)
    end{vmatrix}$$



    Now divide $i$-th row by $a_i$ for $i =1, ldots, n$ to obtain



    begin{align}
    0 &= begin{vmatrix} 1-lambda & 1 & cdots & 1 \
    1 & 1-lambda & cdots & 1 \
    vdots & vdots & ddots & vdots \
    1 & 1 & cdots & 1-lambda
    end{vmatrix} \
    &= begin{vmatrix} 1-lambda & 1 & cdots & 1 \
    lambda & -lambda & cdots & 0 \
    vdots & vdots & ddots & vdots \
    lambda & 0 & cdots & -lambda
    end{vmatrix} \
    &= begin{vmatrix} n-lambda & 1 & cdots & 1 \
    0 & -lambda & cdots & 0 \
    vdots & vdots & ddots & vdots \
    0& 0 & cdots & -lambda
    end{vmatrix} \
    &= (n-lambda)(-lambda)^{n-1}
    end{align}



    so the eigenvalues are $0$ and $n$.






    share|cite|improve this answer











    $endgroup$









    • 1




      $begingroup$
      I think it is wrong. let a1=1 and a2 =1 then M is [[1,1],[1,1]]. but the eigenvalues of M are 0 and 2
      $endgroup$
      – Peyman mohseni kiasari
      Dec 20 '18 at 20:10










    • $begingroup$
      @Peyman Whoops, you are right. The eigenvalues are $0$ and $n$.
      $endgroup$
      – mechanodroid
      Dec 20 '18 at 20:15














    2












    2








    2





    $begingroup$

    To find the eigenvalues, we are calculating the zeroes of the characteristic polynomial of $M$.



    $$0= det(M - lambda I) = begin{vmatrix} 1-lambda &frac{a_1}{a_2} & cdots & frac{a_1}{a_n} \
    frac{a_2}{a_1} & 1-lambda & cdots & frac{a_2}{a_n} \
    vdots & vdots & ddots & vdots \
    frac{a_n}{a_1} & frac{a_n}{a_2} & cdots & 1-lambda
    end{vmatrix}$$



    Since $a_1, ldots, a_n ne 0$, we can multiply $j$-th column by $a_j$ for $j = 1, ldots, n$ to obtain:



    $$0 = begin{vmatrix} a_1(1-lambda) & a_1 & cdots & a_1 \
    a_2 & a_2(1-lambda) & cdots & a_2 \
    vdots & vdots & ddots & vdots \
    a_n & a_n & cdots & a_n(1-lambda)
    end{vmatrix}$$



    Now divide $i$-th row by $a_i$ for $i =1, ldots, n$ to obtain



    begin{align}
    0 &= begin{vmatrix} 1-lambda & 1 & cdots & 1 \
    1 & 1-lambda & cdots & 1 \
    vdots & vdots & ddots & vdots \
    1 & 1 & cdots & 1-lambda
    end{vmatrix} \
    &= begin{vmatrix} 1-lambda & 1 & cdots & 1 \
    lambda & -lambda & cdots & 0 \
    vdots & vdots & ddots & vdots \
    lambda & 0 & cdots & -lambda
    end{vmatrix} \
    &= begin{vmatrix} n-lambda & 1 & cdots & 1 \
    0 & -lambda & cdots & 0 \
    vdots & vdots & ddots & vdots \
    0& 0 & cdots & -lambda
    end{vmatrix} \
    &= (n-lambda)(-lambda)^{n-1}
    end{align}



    so the eigenvalues are $0$ and $n$.






    share|cite|improve this answer











    $endgroup$



    To find the eigenvalues, we are calculating the zeroes of the characteristic polynomial of $M$.



    $$0= det(M - lambda I) = begin{vmatrix} 1-lambda &frac{a_1}{a_2} & cdots & frac{a_1}{a_n} \
    frac{a_2}{a_1} & 1-lambda & cdots & frac{a_2}{a_n} \
    vdots & vdots & ddots & vdots \
    frac{a_n}{a_1} & frac{a_n}{a_2} & cdots & 1-lambda
    end{vmatrix}$$



    Since $a_1, ldots, a_n ne 0$, we can multiply $j$-th column by $a_j$ for $j = 1, ldots, n$ to obtain:



    $$0 = begin{vmatrix} a_1(1-lambda) & a_1 & cdots & a_1 \
    a_2 & a_2(1-lambda) & cdots & a_2 \
    vdots & vdots & ddots & vdots \
    a_n & a_n & cdots & a_n(1-lambda)
    end{vmatrix}$$



    Now divide $i$-th row by $a_i$ for $i =1, ldots, n$ to obtain



    begin{align}
    0 &= begin{vmatrix} 1-lambda & 1 & cdots & 1 \
    1 & 1-lambda & cdots & 1 \
    vdots & vdots & ddots & vdots \
    1 & 1 & cdots & 1-lambda
    end{vmatrix} \
    &= begin{vmatrix} 1-lambda & 1 & cdots & 1 \
    lambda & -lambda & cdots & 0 \
    vdots & vdots & ddots & vdots \
    lambda & 0 & cdots & -lambda
    end{vmatrix} \
    &= begin{vmatrix} n-lambda & 1 & cdots & 1 \
    0 & -lambda & cdots & 0 \
    vdots & vdots & ddots & vdots \
    0& 0 & cdots & -lambda
    end{vmatrix} \
    &= (n-lambda)(-lambda)^{n-1}
    end{align}



    so the eigenvalues are $0$ and $n$.







    share|cite|improve this answer














    share|cite|improve this answer



    share|cite|improve this answer








    edited Dec 20 '18 at 20:13

























    answered Dec 20 '18 at 20:07









    mechanodroidmechanodroid

    28.9k62648




    28.9k62648








    • 1




      $begingroup$
      I think it is wrong. let a1=1 and a2 =1 then M is [[1,1],[1,1]]. but the eigenvalues of M are 0 and 2
      $endgroup$
      – Peyman mohseni kiasari
      Dec 20 '18 at 20:10










    • $begingroup$
      @Peyman Whoops, you are right. The eigenvalues are $0$ and $n$.
      $endgroup$
      – mechanodroid
      Dec 20 '18 at 20:15














    • 1




      $begingroup$
      I think it is wrong. let a1=1 and a2 =1 then M is [[1,1],[1,1]]. but the eigenvalues of M are 0 and 2
      $endgroup$
      – Peyman mohseni kiasari
      Dec 20 '18 at 20:10










    • $begingroup$
      @Peyman Whoops, you are right. The eigenvalues are $0$ and $n$.
      $endgroup$
      – mechanodroid
      Dec 20 '18 at 20:15








    1




    1




    $begingroup$
    I think it is wrong. let a1=1 and a2 =1 then M is [[1,1],[1,1]]. but the eigenvalues of M are 0 and 2
    $endgroup$
    – Peyman mohseni kiasari
    Dec 20 '18 at 20:10




    $begingroup$
    I think it is wrong. let a1=1 and a2 =1 then M is [[1,1],[1,1]]. but the eigenvalues of M are 0 and 2
    $endgroup$
    – Peyman mohseni kiasari
    Dec 20 '18 at 20:10












    $begingroup$
    @Peyman Whoops, you are right. The eigenvalues are $0$ and $n$.
    $endgroup$
    – mechanodroid
    Dec 20 '18 at 20:15




    $begingroup$
    @Peyman Whoops, you are right. The eigenvalues are $0$ and $n$.
    $endgroup$
    – mechanodroid
    Dec 20 '18 at 20:15











    1












    $begingroup$

    We can write your matrix as $M = DJD^{-1}$, where
    $$
    D = pmatrix{a_1\ & ddots \ && a_n}, quad
    J = pmatrix{1 & cdots & 1\ vdots & ddots & vdots \ 1 & cdots & 1}
    $$

    So, $M$ is similar to $J$. $J$ is a rank $1$ symmetric matrix, so it's only non-zero eigenvalue will be $operatorname{tr}(J) = n$.



    We could also recognize that $M$ has rank $1$ as Robert did in his comment.






    share|cite|improve this answer









    $endgroup$


















      1












      $begingroup$

      We can write your matrix as $M = DJD^{-1}$, where
      $$
      D = pmatrix{a_1\ & ddots \ && a_n}, quad
      J = pmatrix{1 & cdots & 1\ vdots & ddots & vdots \ 1 & cdots & 1}
      $$

      So, $M$ is similar to $J$. $J$ is a rank $1$ symmetric matrix, so it's only non-zero eigenvalue will be $operatorname{tr}(J) = n$.



      We could also recognize that $M$ has rank $1$ as Robert did in his comment.






      share|cite|improve this answer









      $endgroup$
















        1












        1








        1





        $begingroup$

        We can write your matrix as $M = DJD^{-1}$, where
        $$
        D = pmatrix{a_1\ & ddots \ && a_n}, quad
        J = pmatrix{1 & cdots & 1\ vdots & ddots & vdots \ 1 & cdots & 1}
        $$

        So, $M$ is similar to $J$. $J$ is a rank $1$ symmetric matrix, so it's only non-zero eigenvalue will be $operatorname{tr}(J) = n$.



        We could also recognize that $M$ has rank $1$ as Robert did in his comment.






        share|cite|improve this answer









        $endgroup$



        We can write your matrix as $M = DJD^{-1}$, where
        $$
        D = pmatrix{a_1\ & ddots \ && a_n}, quad
        J = pmatrix{1 & cdots & 1\ vdots & ddots & vdots \ 1 & cdots & 1}
        $$

        So, $M$ is similar to $J$. $J$ is a rank $1$ symmetric matrix, so it's only non-zero eigenvalue will be $operatorname{tr}(J) = n$.



        We could also recognize that $M$ has rank $1$ as Robert did in his comment.







        share|cite|improve this answer












        share|cite|improve this answer



        share|cite|improve this answer










        answered Dec 20 '18 at 20:26









        OmnomnomnomOmnomnomnom

        129k794188




        129k794188






























            draft saved

            draft discarded




















































            Thanks for contributing an answer to Mathematics Stack Exchange!


            • Please be sure to answer the question. Provide details and share your research!

            But avoid



            • Asking for help, clarification, or responding to other answers.

            • Making statements based on opinion; back them up with references or personal experience.


            Use MathJax to format equations. MathJax reference.


            To learn more, see our tips on writing great answers.




            draft saved


            draft discarded














            StackExchange.ready(
            function () {
            StackExchange.openid.initPostLogin('.new-post-login', 'https%3a%2f%2fmath.stackexchange.com%2fquestions%2f3047908%2fwhat-are-the-eigenvalues-of-the-given-matrix-m%23new-answer', 'question_page');
            }
            );

            Post as a guest















            Required, but never shown





















































            Required, but never shown














            Required, but never shown












            Required, but never shown







            Required, but never shown

































            Required, but never shown














            Required, but never shown












            Required, but never shown







            Required, but never shown







            Popular posts from this blog

            Plaza Victoria

            In PowerPoint, is there a keyboard shortcut for bulleted / numbered list?

            How to put 3 figures in Latex with 2 figures side by side and 1 below these side by side images but in...